PT32.S3.G2 - Will someone come lšŸ‘€k at my question about this game? It's so simple but i'm stumped

markmywordsmarkmywords Core Member
edited February 18 in Logic Games 261 karma

I posted a q on the thread for PT32 S3 G2 here https://7sage.com/lsat_explanations/lsat-32-section-3-game-2/

It asks
"Any one of the following could be true about the organizer's selection of works EXCEPT"
It's an in and out game. The answer choices give a partial list of what's in and are asking if the combination could be true. I'm confused because [I thought] when they ask what could be in, we're looking at a comprehensive list, rather than partial list--so it really threw me. I scavenged the comment section and no one else seemed to get tripped up so clearly this is me šŸ˜…
-Can someone explain how you knew the AC's were partial?
-Would you say this is a unique q or naw common get a grip

Reply here or directly on the question thread! tysm!

Comments

  • Xavier166Xavier166 Alum Member
    153 karma

    Because the maximum that must be selected is 5 and answer choice A has only four selected. According to answer choice A no Russian novels are selected and exactly one play is selected, with that information you have no choice but to select all 3 French novels since the minimum novels selected is 3 and one play, does not matter which play either way it does not meet the minimum criteria. Be sure to pay attention to words like "complete, exactly" Hope that made sense.

  • markmywordsmarkmywords Core Member
    261 karma

    @Xavier158 That was so obvious. I have no idea why I got so stuck on it. I think i've just programmed by brain to see patterns and for whatever reason this one wasn't downloaded yet. Thanks for taking the time, 'preciate it so much!!!

Sign In or Register to comment.